FINAL

Pataasin ang iyong marka sa homework at exams ngayon gamit ang Quizwiz!

A nurse is caring for a client who will begin using transdermal nitroglycerin to treat angina pectoris. When speaking to the client about the drug, which of the following instructions should the nurse include? (Select all) "Apply a new patch at the onset of anginal pain." "Apply the patch to dry skin and cover the area with plastic wrap." "Apply the patch to a hairless area and rotate sites." "Apply a new patch when you start your day." "Remove patches for 10 to 12 hours each day."

"Apply the patch to a hairless area and rotate sites." "Apply a new patch when you start your day." "Remove patches for 10 to 12 hours each day."

9. Which statement by a patient taking depot medroxyprogesterone acetate (DMPA) indicates to the nurse a need for further teaching? "This drug keeps me from ovulating." "I should not take this drug for longer than 2 years." "One injection should protect me from getting pregnant for about 3 months." "I should be able to get pregnant within 6 months of stopping the drug."

"I should be able to get pregnant within 6 months of stopping the drug."

A nurse is caring for a client who takes low dose aspirin to prevent cardiovascular events. The client asks the nurse about taking ibuprofen to treat rheumatoid arthritis. Which of the following responses should the nurse make? "Ibuprofen will increase your risk for developing salicylism." "Ibuprofen will reduce the cardioprotective effects of low-dose aspirin." "Low-dose aspirin will reduce the anti-inflammatory effects of ibuprofen." "Low-dose aspirin will reduce the analgesic effects of ibuprofen."

"Ibuprofen will reduce the cardioprotective effects of low-dose aspirin."

A nurse is teaching a client who has a new prescription for quinidine. Which of the following statements should the nurse include? "Monitor your pulse rate and report changes." "Remain upright for 30 minutes after taking this medication." "This medication can decrease digoxin levels." "This medication can cause urinary incontinence."

"Monitor your pulse rate and report changes."

5. How should the nurse respond to a patient treated with malathion who asks whether a blow dryer can be used to dry the hair? "Yes, this will also help kill the lice." "Yes, this shortens the time that the drug must be left on the skin." "No, the hair must dry naturally for the medication to work." "No, the medication is flammable, so avoid any heat near the head."

"No, the medication is flammable, so avoid any heat near the head."

7. What is the best response by a nurse caring for a patient taking pyrantel for a pinworm infestation and complaining of insomnia and headaches? "This is a major side effect, and you need to stop the medication." "Take the medication at night because this will decrease the side effects." "Take the medication in the morning so you will improve your ability to sleep." "Discontinue the medication; these side effects should not be occurring."

"Take the medication in the morning so you will improve your ability to sleep."

A patient contacts a clinic nurse to determine the proper action after she forgot to take her oral contraceptive [Ortho Tri-Cyclen] for the past 2 days during the first week of a 28-day regimen. Which response by the nurse is most appropriate? "Take the omitted two doses together with the next dose." "Take two doses per day on the following 2 days." "Stop taking the oral contraceptive until menstruation occurs." "Take a dose now and continue with the scheduled doses."

"Take two doses per day on the following 2 days."

10. What advice should the nurse give to the patient who has been treated for lice and needs to get rid of the nits? (Select all that apply.) "Shave your head to get rid of the nits." "Cut your hair very short so you can brush the nits out." "Use a small pair of tweezers to pick the nits off the hair." "Use a nit comb to remove the nits."

"Use a small pair of tweezers to pick the nits off the hair." "Use a nit comb to remove the nits."

6. How should the nurse respond to the patient who used spinosad 3 days ago and wants to repeat the treatment because lice are still present? "Repeat treatment anytime after the third day after initial treatment." "Wait 1 more day before repeating the therapy." "Wait 3 more days before repeating the therapy." "Wait 7 days before repeating the therapy."

"Wait 7 days before repeating the therapy."

A patient is taking nitrofuantion for a UTI. The nurse should tell the client to report which of the following adverse effects? Constipation Dark brown urine Cough Tremors

Cough

A nurse should questions the use of conjugated equine estrogen by a client who has a history of which of the following? A. Osteoporosis B. Peptic ulcer disease C. Rheumatoid arthritis D. Blood Clots

D. Blood Clots

5. Which patient taking tinidazole should the nurse monitor closely? A 65 year old with cataracts A 60 year old with hearing loss A 40 year old with a seizure disorder A 50 year old with well-controlled diabetes

A 40 year old with a seizure disorder

2.The nurse instructs the patient being treated for lice with permethrin to leave the medication on the scalp for how long? 1 hour 3 hours 30 minutes 10 minutes

10 minutes

4.The patient being treated with malathion for head lice should be instructed by the nurse to leave the medication on the scalp for how long? 10 minutes 5 hours 12 hours 24 hours

12 hours

3.The nurse instructs the patient being treated for scabies to leave the medication on the skin for how long? 1 hour 5 hours 14 hours 20 hours

14 hours

9. The nurse informs the patient beginning therapy for neurosyphilis that the treatment will last how long? 2 weeks 12 weeks 4 to 6 weeks 6 to 8 weeks

2 weeks

A nurse is caring for a client who is about to begin taking epoetin. An increase in which of the following laboratory values should indicate to the nurse that the therapy us effective? PT WBC Hgb Platelets

Hgb

A 28-year-old with seizure disorder has a vagus nerve stimulator implanted to help control seizure activity. Which result will the nurse expect to occur? Decreased cardiac conduction Increased speed of cardiac cycle Increased cardiac contractility Increased speed of cardiac cycle

Decreased cardiac conduction

A nurse is caring for a client who is taking ferrous sulfate to treat iron deficiency anemia and develops iron toxicity. Which of the following drugs should the nurse expect to use to treat this complication? Flumazenil Acetylcysteine Naloxone Deferoxamine

Deferoxamine

A nurse is caring for a patient who is prescribed acyclovir IV to treat a viral infection. The nurse should be mindful fo which of the following conditions? Heart failure Dehydration Asthma Tinnitus

Dehydration

On average, the kidneys receive approximately _____ of the cardiac output. 10-14% 15-19% 20-25% 26-35%

20-25%

10. How would the nurse best describe botulinum toxin? It is a lethally potent poison. It is treated with aggressive antibiotic therapy. It is a mild poison that produces few symptoms. It produces illness only in the young, elderly, or immune compromised.

It is a lethally potent poison.

9. The nurse should instruct the patient being treated for head lice with lindane to leave the medication on the scalp for how long? 1 minute 4 minutes 10 minutes 15 minutes

4 minutes

A nurse is providing teaching to a client about sildenafil to treat erectile dysfuntion. Which of the following instructions should the nurse include? A. Take the drug with a glass of grapefruit juice B. Do not take the drug with high-fat foods C. Increase intake of foods rich in vitamin k D. avoid aged cheeses and red wine

B. Do not take the drug with high-fat foods

4. The nurse identifies which medication as a first-line drug treatment of mild to moderate acne? Tetracycline Isotretinoin Tretinoin Benzoyl peroxide

Benzoyl peroxide

The nurse identifies which medication as a first-line drug treatment of mild to moderate acne? Tetracycline Isotretinoin Tretinoin Benzoyl peroxide

Benzoyl peroxide

10. The nurse identifies which reaction in the patient being treated for onchocerciasis who experiences pruritus, rash, fever, lymph node tenderness, and bone and joint pain? Stevens-Johnson syndrome Anaphylactic reaction Mazzotti reaction Gray's reaction

Mazzotti reaction

7.The nurse should teach a patient being treated for scabies with crotamiton to leave the medication on the skin for ________ hours after the second application. 10 18 30 48

48

8. In providing teaching to a patient receiving levonorgestrel, the nurse states that this contraceptive is sufficient for up to what period? 1 year 3 years 5 years 7 years

5 years

8. The nurse understands that which patient is not a candidate for the use of lindane to treat lice? A 20 year old who is having surgery A 30 year old with a history of seizures A 10 year old with diabetes A 13 year old with asthma

A 30 year old with a history of seizures

A nurse is caring for a group of postoperative clients. The nurse should identify that morphine is contraindicated for which of the following clients? A client who had a mastectomy A client who had a knee arthroplasty A client who had a colectomy A client who had a cholecystectomy

A client who had a cholecystectomy

During the relative refractory period of the action potential: A stronger than normal stimulus can elicit an action potential A second action potential cannot be elicited, even with strong stimulus An even weaker stimulus can elicit an new action potential. The weakest stimuli can elicit a new action potential.

A second action potential cannot be elicited, even with strong stimulus

A nurse is caring for a client who is about to begin taking finasteride to treat benign prostatic hypertrophy. The nurse should explain to the client the need to monitor which of the following laboratory values? A. Prostate-specific antigen (PSA) B. BUN C. Creatine Phosphkinase (CPK) D. ALT

A. Prostate-specific antigen (PSA)

A nurse is providing teaching to a client who is taking bethanechol for urinary retention. Which of the following instructions should the nurse include (select all that apply) A. Avoid driving and activities that require alertness B. Increase fluid intake C. take the drug with food D. Stay close to a bathroom after taking the drug E. Increase weight-bearing activities

A. Avoid driving and activities that require alertness B. Increase fluid intake D. Stay close to a bathroom after taking the drug

A nurse is providing safety education to a client who is scheduled to begin treatment with tamsulosin for benign prostatic hypertrophy (BPH). Which of the following instructions should the nurse include in the teaching? A. Change positions slowly B. Crush the medication C. Take the medication on an empty stomach D. Report increased urinary output immediately

A. Change positions slowly

A provider has prescribed a dinoprostone vaginal insert to promote cervical ripening in a client who is at 40wks of gestation. When administering dinoprostone, which of the following actions should the nurse take? (Select all that apply.) A. Encourage the client to urinate prior to insertion B. Remove the pouch when active labor begins C. Monitor for uterine hyperstimulation D. Encourage the client to walk after insertion E. Monitor the clients temperature

A. Encourage the client to urinate prior to insertion B. Remove the pouch when active labor begins C. Monitor for uterine hyperstimulation E. Monitor the clients temperature

A nurse should question the use of ethinyl estradiol and drospirenone by a client who has renal disease due to the increased risk of thich of the following? A. Hyperkalemia B. Hyponatremia C. Hypoglycemia D. Respiratory alkalosis

A. Hyperkalemia

A nurse is providing teaching to a clent about using leuprolide to treat endometriosis. Which of the following instructions should the nurse include? A. Perform weight-bearing activities B. Wear sunscreen or protective clothing C. Take the drug with food to increase absorption D. Avoid drinking grapefruit

A. Perform weight-bearing activities

A nurse is caring for a client who has a new prescription for transdermal conjugated estrogen and medroxyprogesterone to treat postmenopausal symptoms. The nurse shoudl explain to the client that this drug combination includes which of the following therapeutic effects? (Select all that apply.) A. Reduces the risk of ovarian cancer B. Relieves hot flashes C. Prevents osteoporosis D. Reduces the risk of breast cancer E. Reduces the risk of thromboembolism

A. Reduces the risk of ovarian cancer B. Relieves hot flashes C. Prevents osteoporosis

A nurse is teaching a client about receiving leuprolide to treat endometriosis. The nurse should instruct the client to expect which of the following side effects? A. Vaginal dryness B. Hypertension C. Blurred vision D. Orthostatic hypotension

A. Vaginal dryness

A 17-year-old male patient asks about treatment options for Chlamydia trachomatis infection. Which response by the nurse is appropriate? A."Chlamydia is bacterial and is treated with azithromycin [Zithromax]." B."This infection is treatable but is best prevented by vaccination." C."The disease is viral and will be treated with valacyclovir [Valtrex]." D."The infection is fungal, and metronidazole [Flagyl] will be prescribed."

A."Chlamydia is bacterial and is treated with azithromycin [Zithromax]."

2.Which male reproductive disorder is associated with mumps? A.Orchitis B.Balanitis C.Urethritis D.Prostatitis

A.Orchitis

4. During therapy with amphotericin B, the nurse confirms that what laboratory values are measured every 3 to 4 days? Complete blood cell count (CBC) Plasma creatinine Platelet level Urine osmolality

Plasma creatinine

A nurse is caring for a client who is taking acetaminophen at regular intervals for mild discomfort. The nurse should tell the client to report which of the following early indications of acetaminophen toxicity? (Select all) Diaphoresis Palpitations Shortness of breath Nausea Diarrhea

Diaphoresis Nausea Diarrhea

A nurse is teaching a client who is starting treatment with warfarin. The nurse should plan to include information on which of the following topics to promote the effectiveness of the drug? Sleep modifications Fluid modifications Driving modifications Dietary modifications

Dietary modifications

6. Nondrug procedures for poisoning treatment are most effective when what occurs? (Select all that apply.) Binding of toxicants to plasma protein is low The health care team knows the time before injestion Blood levels of the toxicants are high Urine volume remains greater than 30 mL/hr

Binding of toxicants to plasma protein is low Blood levels of the toxicants are high

A 54-year-old female is diagnosed with nephritic syndrome. Which of the following is a common symptom of this disease? Proteinuria Dysuria Hematuria Oliguria

Hematuria

9. The nurse should monitor which laboratory value(s) in patients taking flucytosine? Hemoglobin and hematocrit (H & H) Urine osmolality Platelets and leukocyte count Blood urea nitrogen (BUN) and creatinine

Platelets and leukocyte count

A HCP is caring for a pt who is taking warfarin (Coumadin) and is about to begin taking trimethoprim/sulfamethoxazole (bactrim) to treat a UTI. The HCP should question the drug regimen bc taking these two drugs concurrently can increase the risk for which of the following? Bleeding Thrombosis ECG changes Ototoxicity

Bleeding

An older male presents with flank pain and polyuria. Tests reveal that he has an enlarged prostate. Which type of renal failure is this patient at risk for? Postrenal Prerenal Extrarenal Intrarenal

Postrenal

Blood flow is affected by: (select all that apply) blood vessel diameter. blood viscosity. blood pressure. blood composition. blood vessel length.

blood vessel diameter. blood viscosity. blood pressure. blood vessel length.

10. The nurse recognizes that ________ is the greatest adverse effect associated with depot medroxyprogesterone acetate (DMPA). bone loss hypertension dysmenorrhea cervical infection

bone loss

What term is used to identify the period that follows depolarization of the myocardium and represents a period during which no new cardiac potential can be propagated? Absolute refractory Hyperpolarization Threshold Resting

Absolute refractory

A nurse is preparing to administer amphotericin B IV to a client who has a systemic fungal infection. Which of the following drugs should the nurse prepare to administer prior to the infusion to minimize adverse reactions? Select all Aspirin Hydrocortisone Acetaminophen Diphenhydramine Ibuprofen

Acetaminophen Diphenhydramine

1. The nurse would administer what pretreatment for a patient receiving amphotericin B intravenously? Acetaminophen and diphenhydramine Ibuprofen and diphenhydramine Dimenhydrinate and dantrolene Meperidine or dantrolene

Acetaminophen and diphenhydramine

A 29-year-old female presents with cloudy urine, flank pain, and hematuria. These signs and symptoms support which diagnosis? Renal calculi Acute cystitis Chronic renal failure Postrenal renal failure

Acute cystitis

A nurse should recognize that milrinone is contraindicated for a client who has which of the following conditions? Acute myocardial infarction Peripheral vascular disease Ulcerative colitis Ventricular tachycardia

Acute myocardial infarction

A nurse is administering epoetin intravenously to a client who has renal failure. Which of the following actions should the nurse take? Shake the vial before using. Administer via IV bolus over 1 to 3 min. Dilute the drug first with D5W. Save the used vial for the next dose.

Administer via IV bolus over 1 to 3 min.

A nurse is discussing the pressure generated at the end of diastole. Which term is the nurse describing? Preload Systemic vascular resistance Afterload Total peripheral resistance

Afterload

3. The nurse should teach a patient taking metronidazole to avoid what? Caffeinated drinks Sugary foods Fruit juices Alcohol

Alcohol

A nurse is reviewing the medical record of a client who reports taking acetaminophen at home. The nurse should identify that which of the following client conditions is contraindication for acetaminophen? Asthma Diabetes mellitus Heart failure Alcohol use disorder

Alcohol use disorder

A nurse is caring for a client who is experiencing an acute ischemic cerebrovascular event due to a thrombus in a cerebral vessel. Which of the following drugs should the nurse expect to administer? Alteplase Aspirin Clopidogrel Heparin

Alteplase

A nurse is caring for a client who is about to begin alteplase therapy to treat pulmonary embolism. Which of the following drugs should the nurse have available in the event of a severe adverse reaction? Vitamin K Aminocaproic acid Protamine Deferoxamine

Aminocaproic acid

What is the name of the enzyme secreted by the juxtaglomerular cells in the kidney that is responsible for vascular tone and renal sodium excretion? Angiotensin I Angiotensinogen Angiotensin II Renin

Angiotensin I

Reabsorption of water in the collecting ducts requires which hormone? Atrial natriuretic factor (ANP) Antidiuretic hormone (ADH) Renin Aldosterone

Antidiuretic hormone (ADH)

A nurse is caring for a client who is taking digoxin and develops changes on their ECG tracing and other manifestations that indicate the client has severe digoxin toxicity. Which of the following drugs should the nurse expect to administer to treat this complication? Acetylcysteine Flumazenil Antigen-binding fragments Deferoxamine

Antigen-binding fragments

Which statement indicates the nurse understands coronary ostia? The coronary ostia (the openings to the coronary arteries) are found in the: Aorta Left ventricle Coronary sinus Inferior vena cava

Aorta

A 67-year-old was previously diagnosed with rheumatic heart disease. Tests now reveal lipoprotein deposition with chronic inflammation that impairs blood flow from the left ventricle into the aorta. Which diagnosis does this history support? Aortic stenosis Mitral regurgitation Mitral stenosis Aortic regurgitation

Aortic stenosis

What is the most prevalent type of helminthic infestation? Ascariasis Enterobiasis Necatoriasis Ancylostomiasis

Ascariasis

A nurse is caring for a child who has a viral infection. The nurse should identify that which of the following drugs can increase the risk of Reye syndrome in children who have viral infections? Butorphanol Acetaminophen Tramadol Aspirin

Aspirin

A nurse is teaching a client who has a new prescription for allopurinol. Which of the following instructions should the nurse include? Avoid driving or activities that require mental alertness. Avoid crushing the tablets. Limit fluid intake during therapy. Limit potassium while taking allopurinol.

Avoid driving or activities that require mental alertness.

A nurse is caring for a client who has streptococcal pharyngitis and is allergic to penicillin. The nurse should recognize that which of the following drugs can be safely administer to this client? Nafcillin Azithromycin Cephalexin Amoxicillin/clavulanic acid

Azithromycin

5. What is the method of transmission of bubonic plague? It is transmitted by contact with infected lesions. It is transmitted only by infected fleas. It is carried from person to person through oral secretions. It is transmitted from animals to people by fecal contamination.

It is transmitted only by infected fleas.

A nurse is caring for a male client who has hyperthyroidism and is not responding to treatment. The nurse should identify that hyperthyroidism is a contraindication for which of the following drugs? A. Sildenafil B. Bethanechol C. Tamsulosin D. Finasteride

B. Bethanechol

A nurse is teaching a client who has a new prescription for transdermal testosterone gel to treat delayed puberty. Which of the following instructions should the nurse include? (select all that apply) A.Apply the gel to the scrotum B. Cover the area of application w/ clothing C. wash your hands after applying the gel D. Do not shower for sever hours after applying the gel E. apply the gel to the gums above an upper incisor

B. Cover the area of application w/ clothing C. wash your hands after applying the gel D. Do not shower for sever hours after applying the gel

A nurse is assessing a client who has a new prescription of oxybutynin to treat neurogenic bladder. Which of the following assessment findings should the nurse identify as an adverse effect of the drug? (Select all that apply) A. Diaphoresis B. Dilated pupils C. Dizziness D. Distended bladder E. Fever

B. Dilated pupils C. Dizziness D. Distended bladder E. Fever

A nurse is caring for a client who has benign prostatic hypertrophy and is taking tamsulosin. The nurse should question the use of the drug if the client also has which of the following? A. A seizure disorder for which he takes carbamazepine B. Erectile dysfuntion for which he take sildenafil C. Diabetes mellitus for which he takes glyburide D. Angle-closure glaucoma for which he take pilocarpine

B. Erectile dysfuntion for which he take sildenafil

A nurse is assessing a client who has a new prescription for the combination oral contraceptive ethinyl estradiol and drospirenone. Which of the following findings should the nurse identify as a contraindication to use of this medication? A. Drinks a glass of wine each night B. Smokes half a pack of cigarettes a day C. Runs marathons regularly D. Drives a school bus

B. Smokes half a pack of cigarettes a day

Which is(are) the preferred drug(s) for the treatment of gonorrhea? A.Doxycycline and azithromycin B.Ceftriaxone C.Metronidazole D.Acyclovir [Zovirax]

B.Ceftriaxone

A client is taking acyclovir to treat a herpes simplex virus. Which of the following lab values should the nurse monitor? Prothrombin time Hct BUN Aspartate aminotransferase

BUN

For an infection to progress to septic shock, what must occur? Bacteria must enter the bloodstream. The individual must be immunosuppressed. The myocardium must be impaired. The infection must be gram negative.

Bacteria must enter the bloodstream.

What is the name of the reflex that results when BP decreases and pressoreceptors in the carotid bodies and aortic arch are stimulated?

Bainbridge

A nurse is caring for a client who has a new prescription for prednisone for long term treatment of rheumatoid arthritis. The nurse should monitor the client for which of the following adverse drug reactions? Pulmonary embolism Hepatitis Bone loss Breast cancer

Bone loss

A patient taking penicillin V three times a day reports abdominal cramping with bloody diarrhea. Which of the following instructions should the nurse give?

Bring in a stool sample for testing

A 15-year-old male who is allergic to peanuts eats a peanut butter cup. He then goes into anaphylactic shock. Which assessment findings will the nurse assess for? Bronchoconstriction, hives or edema, and hypotension Bradycardia, decreased arterial pressure, and oliguria Hypertension, anxiety, and tachycardia Fever, hypotension, and erythematous rash

Bronchoconstriction, hives or edema, and hypotension

What structure conducts action potentials down the atrioventricular septum? Bundle of His Bachmann bundle Atrioventricular node Sinoatrial node

Bundle of His

A patient who is taking tetracycline orally to treat a chlamydia infection contacts the health care professional to report severe blood tinged diarrhea. The nurse should suspect the client is experiencing which of the following? Hemorrhoids Clostridium difficile-associated diarrhea Diverticular disease Small bowel obstruction

C diff

A nurse is caring for a client who missed a dose of her oral contraceptive. Which of the following instructions should the nurse give the client? A. Take a pregnancy test B. Double up on pills for remainder of the pack C. Take the missed dose as soon as possible D Stop the oral contraceptive and use a different contraceptive

C. Take the missed dose as soon as possible

A 54-year right-handed female with recent onset of sinusitis develops photophobia and nuchal rigidity. On exam, she has a positive Kernig's and Brudzinski's sign. She most likely has which of the following? A. Brain abscess B. Encephalitis C. Bacterial meningitis D. Spinal cord abscess

C. Bacterial meningitis

A nurse is caring for a client who received human menopausal gonadotropin (hMG). Which of the following findings should the nurse expect? A. Prolactin secretion B. Estrogen release C. Follicular maturation. D. Oxytocin secretion

C. Follicular maturation.

A nurse is teaching a client about conjugated estrogen and medroxyprogesterone. Which of the following instructions should the nurse include? A. Monitor for rectal bleeding B. Avoid drinking alcoholic beverages C. Have blood pressure checked regularly D. sit up for 30min after taking the drug

C. Have blood pressure checked regularly

A nurse is caring for a client who is about to begin clomiphene therapy to treat infertility. The nurse should monitor the client for which of the following adverse effects? A. Breast cancer B. Deep-vein thrombosis C. Ovarian hyperstimulation syndrome D. Restless leg syndrome

C. Ovarian hyperstimulation syndrome

A nurse is caring for a client who has a new prescription for conjugated equine estrogen. The nurse should instruct the client to report which of the following indication of a serious adverse reaction? A. Hot flashses B. urinary tract infection C. Vaginal bleeding D. Bone pain

C. Vaginal bleeding

A nurse is preparing to administer finasteride to a client who has benign prostatic hypertrophy (BPH). Which of the following precautions should the nurse plan to take when administering this drug? A. Refrain from crushing the tablet B. Withhold the medication if the client doesnt eat C. Wear gloves when handling the tablet D. Monitor heart rate prior to administration

C. Wear gloves when handling the tablet

A nurse is monitoring a client following ferrous sulfate administration. The nurse should monitor the client for which of the following adverse effects? Phlebitis Dark, orange-colored stools Constipation Injection site pain

Constipation

A patient with asthma is prescribed albuterol [Proventil], 2 puffs 3 times a day. The nurse should teach the patient to do what? A.Rinse the mouth after taking the prescribed dose. B.Take an extra dose if breathing is compromised. C.Wait 1 minute between puffs from the inhaler. D.Take adequate amounts of calcium and vitamin D.

C.Wait 1 minute between puffs from the inhaler.

What is the most common type of renal stone composed of? Uric acid Cystine Struvite Calcium

Calcium

A patient who is taking amoxicillin to treat a respiratory infection contacts the health care professional to report a rash and wheezing. Which of the following instructions should the health care professional provide? Wait 1 hr and contact the provider if there is no improvement. Skip today's dose of amoxicillin and resume taking the drug tomorrow. Call emergency services immediately. Take an NSAID to reduce skin and airway inflammation.

Call emergency services immediately.

When administering erythromycin to a patient with cystic acne, a nurse should monitor for which of the following adverse effects? Hypothermia Blurred vision Constipation Cardiac dysrhythmias

Cardiac dysrhythmias

A nurse observes a cardiologist multiplying the heart rate by stroke volume. What is the cardiologist measuring? Cardiac output Vascular resistance Ejection fraction Preload

Cardiac output

A primary care provider is considering the various pharmalogic options for a patient who has a gynecologic infection and a history of alcohol use disorder. Which of the following medications can cause a reaction similar to disulfiram (antabuse) if the patient drinks alcohol while taking it? Select all Nitrofurantoin Amoxicillin Aztreonam Cefotetan Metronidazole

Cefotetan Metronidazole

4. What does the nurse identify as the preferred treatment for adults with gonorrhea? Ceftriaxone Interferon Vancomycin Erythromycin

Ceftriaxone

5. What does the nurse recognize as the recommended therapy for neonatal gonococcal ophthalmia? Interferon Vancomycin Doxycycline Ceftriaxone

Ceftriaxone

9. The nurse identifies what as the most common treatment for poisoning from heavy metals? Chelation Hemodialysis Plasmapheresis Peritoneal dialysis

Chelation

A nurse is caring for a client who has a new prescription for celecoxib. The nurse should tell the client to report which of the following adverse drug reactions? Tinnitus Chest pain Constipation Diaphoresis

Chest pain

Which structures act as anchors for the atrioventricular valves? Chordae tendineae Great vessels Coronary ostia Trabeculae carneae

Chordae tendineae

An older adult is diagnosed with cerebral aneurysm. Where does the nurse suspect the cerebral aneurysm is located? Circle of Willis Vertebral arteries Basilar artery Carotid arteries

Circle of Willis

A nurse is preparing to administer a prescribed dose of desmopressin to a client who has hemophilia A. The client's laboratory results indicate that the client has a sodium level of 130 mEq/L. Which of the following actions should the nurse take? Clarify the prescription with the provider. Administer the drug with an analgesic. Administer the required dose orally. Assess factor IX levels.

Clarify the prescription with the provider.

A nurse is caring for a client who has chronic stable angina. The nurse should identify that which of the following drugs inhibits the action of adenosine diphosphate receptors (ADP) on platelets and can be prescribed to reduce the client's risk for myocardial infarction? Clopidogrel Heparin Warfarin Alteplase

Clopidogrel

During the isovolumic phase of ventricular systole, the aortic valve is normally open or closed?

Closed

A patient has a severe kidney obstruction leading to removal of the affected kidney. Which of the following would the nurse expect to occur? Atrophy of the remaining kidney Dysplasia in the remaining kidney Compensatory hypertrophy of the remaining kidney Renal failure

Compensatory hypertrophy of the remaining kidney

The region of the kidneys that contains the glomeruli is called the: Cortex Medulla Pyramids Columns

Cortex

A nurse is caring for a client who is about to begin therapy with recombinant factor IX to treat hemophilia B. The client asks the nurse about the risk of disease transmission with recombinant factor IX as compared with plasma derived factor IX. The nurse should explain that the recombinant factor IX practically eliminates the risk foe which of the following? HIV Cytomegalovirus Creutzfeldt-Jakob disease Anaphylaxis

Creutzfeldt-Jakob disease

7. Which medications should be avoided in a patient taking itraconazole? (Select all that apply.) Cyclosporine Warfarin Quinidine Digoxin Insulin

Cyclosporine Warfarin Quinidine Digoxin

After administration of Terbutaline to a client to inhibit preterm labor, the nurse should assess which of the following? A. Peripheral pulses B. Temperature C. Vision D. Heart Rate

D. Heart rate

A nurse is teaching a client who has a new prescription for a testosterone transdermal patch. Which of the following statements made by the client indicates an understanding of the teaching? A. I should have pancreatic function tests obtained while taking this drug B. I should have my white blood cell count checked annually C. I should take off the patch to shower D. I should apply the medication to a different site each time

D. I should apply the medication to a different site each time

A nurse is caring for a client who is about to begin taking cabergoline to treat infertility. The nurse should explain to the client the need to monitor which of the following laboratory values? A. Potassium B. Glucose C. BUN and creatinine D. Prolactin Level

D. Prolactin Level

A nurse is caring for a client who is taking combination oral contraceptive ethinyl estradiol and drospirenone and is about to begin taking rifampin to treat tuberculosis. Which of the following instructions should the nurse give the client due to a possible drug interaction? A. increase the rifampin dose B. increase the oral contraceptive dose C. wait 2hrs between taking each drug D. use additional birth control methods

D. use additional birth control methods

The nurse teaches a patient about Camila. Which statement by the patient requires an intervention by the nurse? A."I might have irregular bleeding while taking this pill." B."These pills do not usually cause blood clots." C."I should take this pill at the same time every day." D."This pill works primarily by preventing ovulation."

D."This pill works primarily by preventing ovulation."

A patient who was taking sulfonamides develops Stevens-Johnson syndrome. Upon assessment, the nurse expects to find what? A.Hypotension B.Bronchospasm C.Temperature of 35.5°C D.Widespread skin lesions

D.Widespread skin lesions

A patient is diagnosed with urinary tract obstruction. While planning care, the nurse realizes that the patient is expected to have hydronephrosis and a decreased glomerular filtration rate caused by: Decreased renal blood flow Dilation of the renal pelvis and calyces proximal to a blockage Decreased peritubular capillary pressure Stimulation of antidiuretic hormone

Dilation of the renal pelvis and calyces proximal to a blockage

A nurse is teaching a client who has a new prescription for prednisone. Which of the following instructions should the nurse include? (Select all) Reduce the dose during periods of stress. Discontinue the drug gradually. Report illness or infection. Increase intake of calcium and vitamin D. Monitor for signs of gastric bleeding.

Discontinue the drug gradually. Report illness or infection. Increase intake of calcium and vitamin D. Monitor for signs of gastric bleeding.

Which renal structures drain directly into the calyces? (select all that apply) Distal tubule Collecting duct Pyramid Renal pelvis Loop of Henle

Distal tubule Collecting duct

A nurse is teaching a client who is taking allopurinol about minimizing adverse effects. Which of the following instructions should the nurse include? Eat a small meal before taking the drug. Suck on hard candy or chew gum. Take a stool softener daily. Avoid the use of NSAIDs.

Eat a small meal before taking the drug.

A nurse is teaching a client about taking ferrous sulfate to treat iron deficiency anemia. Which of the following instructions should the nurse include? (Select All) Eat iron-enriched foods. Spread the dosage across each day. Take the drug on an empty stomach. Report dark green or black stools. Increase dietary fiber intake.

Eat iron-enriched foods. Spread the dosage across each day. Take the drug on an empty stomach. Increase dietary fiber intake.

A nurse is caring for a client who has hemophilia A and is about to begin taking desmopressin to prevent bleeding. The nurse should monitor the client for which of the following adverse reactions? Weight loss Edema Polyuria Bradycardia

Edema

The aorta is composed of a thick tunica media with more elastic or smooth muscle?

Elastic

A nurse is caring for a client who is taking filgrastim to treat neutropenia. The nurse should assess the client for which of the following adverse effects? Dusky nail beds Petechiae Enlarged spleen Swollen calf

Enlarged spleen

6. Which medication may be given routinely at birth by the nurse to protect against neonatal ophthalmia? (Select all that apply.) Vancomycin Silver nitrate Erythromycin Tetracycline

Erythromycin

Which hormone is synthesized and secreted by the kidneys to stimulate bone marrow production of red blood cells? Creatinine Aldosterone Erythropoietin Renin

Erythropoietin

A 25-year-old female is diagnosed with a urinary tract infection. When the nurse checks the culture results, which of the following organisms is most likely infecting her urinary tract? Streptococcus Candida albicans Escherichia coli Chlamydia

Escherichia coli

A HCP is caring for a patient who is about to receive gentamicin to treat a systemic infection. The healthcare professional should question the use of the drug for a pt who is also taking which of the following drugs? Ethacrynic acid Diphenhydramine Acetaminophen Levothyroxine

Ethacrynic acid

A nurse is providing teaching to a client who has a new prescription for losartan to treat hypertension. The nurse should instruct the client that which of the following findings could indicate an adverse reaction to the drug and needs to be reported? Facial edema Sleepiness Peripheral edema Constipation

Facial edema

A nurse is caring for a client who is taking allopurinol to that gout. The nurse should monitor the client for which of the following manifestations of hypersensitivity syndrome? Muscle pain Fever Anxiety Tremors

Fever

A nurse is caring for a client who is about to begin taking aspirin. The nurse should instruct the client to report which of the following manifestations of salicylism? (Select all) Fever Tinnitus Diaphoresis Thrombophlebitis Dizziness

Fever Tinnitus Diaphoresis Dizziness

A patient is diagnosed with renal calculus that is causing a urinary obstruction. Which symptoms would be most likely experienced? Anuria Flank pain Hematuria Pyuria

Flank pain

10. The nurse understands that what is used to treat poisoning with ethylene glycol, the primary ingredient in antifreeze? Fomepizole Hemodialysis Calcium EDTA Sodium bicarbonate

Fomepizole

4.How would the nurse best describe the use of antidotes in treating poisoning? Administer the antidote only after all other measures have failed. For most poisons, no specific antidote exists. Antidotes should be repeated when necessary. Antidotes are widely available and safe to use.

For most poisons, no specific antidote exists.

A 55-year-old presents reporting urinary retention. Tests reveal a lower urinary tract obstruction. Which of the following is of most concern to the nurse? Vesicoureteral reflux and pyelonephritis Formation of renal calculi Glomerulonephritis Increased bladder compliance

Formation of renal calculi

8. How would the nurse best describe the contrast between using peritoneal dialysis and hemoperfusion to treat poisoning? Hemodialysis is more economical. Hemoperfusion is vastly more effective. Hemodialysis is more complex but more effective. Patient preference determines which method is used.

Hemodialysis is more complex but more effective.

A nurse is reviewing the medication record of a client who is receiving alteplase following an acute myocardial infarction (MI). Which of the following medications should the nurse expect the client to be taking in addition to the alteplase? Protamine Desmopressin Ferrous sulfate Heparin

Heparin

8. How would the nurse best describe acute radiation syndrome? Ataxia, confusion, and weakness Gastrointestinal (GI) ulcerations, skin burns, and hair loss Skin rash, lymphedema, and night blindness Constipation, amenorrhea, and urinary retention

Gastrointestinal (GI) ulcerations, skin burns, and hair loss

A 30-year-old White female was recently diagnosed with primary hypertension. She reports that she eats fairly well, usually moderate red meat consumption. She also reports that her father has hypertension as well. A nurse determines which of the following risk factors is most likely associated with this diagnosis? Genetic Age Diet Race

Genetic

If a nurse wants to obtain the best estimate of renal function, which test should the nurse monitor? Circulating antidiuretic hormone (ADH) levels Volume of urine output Glomerular filtration rate (GFR) Urine-specific gravity

Glomerular filtration rate (GFR)

1. Which drugs are common treatments of inflammatory skin conditions? Glucocorticoids Nonsteroidal antiinflammatory drugs (NSAIDs) Antihistamines Topical anesthetics

Glucocorticoids

8.How should the nurse respond to the patient taking diethylcarbamazine for filarial infestation who wants to know why a pretreatment with glucocorticoids is necessary? This will help the medication work more effectively in eliminating the infestation. Diethylcarbamazine will not work without an increased level of glucocorticoids. Glucocorticoids will decrease the incidence of adverse reactions during treatment. Use of glucocorticoids will allow a smaller dosage of the diethylcarbamazine.

Glucocorticoids will decrease the incidence of adverse reactions during treatment.

A nurse is caring for a client who is taking amiodarone to treat atrial fibrillation. Which of the following should the nurse instruct the client to avoid while taking this drug? Grapefruit juice Milk Foods high in vitamin K NSAIDs

Grapefruit juice

Name the cardiac risk factors that are important to assess in PLWH: (Select all that apply). HAART Smoking Dyslipidemia NIDDM

HAART Smoking Dyslipidemia NIDDM

A nurse is caring for a client who is about to begin factory VIII therapy to treat hemophilia A. When administering factor VIII, which of the following actions should the nurse take? Administer the powdered form orally. Premedicate with aspirin. Administer it via rapid IV bolus. Have emergency equipment ready.

Have emergency equipment ready

A nurse is caring for a client who recently started alteplase therapy. The nurse should monitor the client for which of the following adverse effects? Bronchodilation Headache Edema Hypertension

Headache

A nurse is teaching a client who has a new prescription for nitroglycerin. The nurse should instruct the client that which of the following manifestations is a potential adverse effect of this drug? Headache Constipation Hypertension Hyperglycemia

Headache

6. The nurse is teaching a patient taking praziquantel. What side effects should the patient be aware of? (Select all that apply.) Headache Constipation Abdominal discomfort Dizziness

Headache Abdominal discomfort Dizziness

Select all the complications that you should be monitoring for in someone who has Infective Endocarditis (you may chose more than one) Heart failure Stroke Pulmonary embolism Myocardial infarction Renal dysfunction

Heart failure Stroke Pulmonary embolism Myocardial infarction Renal dysfunction

When a staff member asks which of the following substances is actively secreted by the renal tubules, what is the nurse's best response? Hydrogen and potassium Sodium and chlorine Phosphate and calcium Bicarbonate and carbonic acid

Hydrogen and potassium

A nurse is caring for a client who has a new prescription for aliskiren to treat hypertension. The nurse should monitor the client for which of the following findings as an adverse effect of the drug? (Select all) Hyperkalemia Throat swelling Constipation Cough Joint pain

Hyperkalemia Throat swelling Cough

A nurse is caring for a client who has renal failure and is receiving epoetin. The nurse should monitor the client for which of the following adverse effects? Hypertension Muscle pain Edema Dry mouth

Hypertension

A nurse is administering hydrochlorothiazide to a client who has gouty arthritis. The nurse should monitor the client for which of the following findings that indicates an adverse effect of this drug? Hyperuricemia Hypernatremia Hyperkalemia Hypoglycemia

Hyperuricemia

Which of the following is a symptom of chronic renal failure? Hypotension Hypokalemia Hypocalcemia Hypernatremia

Hypocalcemia

A nurse is caring for a client who is taking carvedilol and has a prescription for an oral antidiabetic drug to manage their new diagnosis of type 2 diabetes. By taking both drugs concurrently the nurse should identify that the client is at an increased risk for which of the following conditions? Hyperglycemia Bradycardia Hypotension Hypoglycemia

Hypoglycemia

5. What electrolyte abnormality should the nurse monitor in a patient receiving amphotericin B infusions? Hypomagnesemia Hypernatremia Hypercalcemia Hypokalemia

Hypokalemia

A 35-year-old hypertensive male begins taking a diuretic. Which of the following common side effects of this medication should the nurse monitor? Increased uric acid secretion Hypermagnesemia Hypokalemia Hyponatremia

Hypokalemia

A nurse is assessing a client who is taking a loop diuretic and is experiencing a thready, irregular pulse, orthostatic hypotension, and confusion. The nurse should identify that these manifestations indicate which of the following adverse effects? Hypokalemia Hypoglycemia Hypouricemia Hyponatremia

Hypokalemia

A nurse is caring for a client who currently takes furosemide who has a new prescription for prednisone. The nurse should monitor the client for which of the following manifestations during concurrent use of the two drugs? Hypercalcemia Hypoglycemia Hypothermia Hypokalemia

Hypokalemia

A nurse is caring for a client who has a new prescription for verapamil to treat atrial fibrillation. The nurse should instruct the client to avoid drinking grapefruit juice while taking verapamil because it can cause the client to experience which of the following conditions? Tachycardia Dehydration Diarrhea Hypotension

Hypotension

An 82-year-old female was admitted to the hospital with confusion and severe hypotension. Her body's compensatory mechanisms are increased heart rate, vasoconstriction, and movement of large volumes of interstitial fluid to the vascular compartment. What kind of shock does the nurse suspect the patient is experiencing? Hypovolemic Anaphylactic Septic Neurogenic

Hypovolemic

Damage to the brain stem at the level of the midbrain (involving cranial nerve 3 & 4) will result in which of the following pupillary defects? A. Unequal pupil size B. Pin point pupils C. Reactive pupils D. Small regular pupils

I THINK C. Reactive pupils

3. What is the initial treatment of choice for inhalational anthrax? Intravenous (IV) acyclovir or IV zidovudine Oral fluconazole or oral miconazole Oral levofloxacin or oral amoxicillin and clavulanic acid IV ciprofloxacin or IV doxycycline

IV ciprofloxacin or IV doxycycline

A 45-year-old presents with hypertension, anorexia, nausea and vomiting, and anemia and is diagnosed with chronic renal failure. What is the cause of this patient's anemia? Red blood cells being lost in the urine Inadequate iron absorption in the gut Red blood cells being injured as they pass through the glomerulus Inadequate production of erythropoietin

Inadequate production of erythropoietin

A 42-year-old male is involved in a motor vehicle accident that has resulted in prerenal failure. What is the most likely cause of this patient's condition? Immune complex deposition in the glomerulus Inadequate renal blood flow Obstruction of the proximal tubule Kidney stones

Inadequate renal blood flow

A nurse is teaching a client who has a new prescription for tramadol. Which of the following instructions should the nurse include? (Select all) Increase fiber and fluid intake. Take the drug with food. Avoid driving after taking the drug. Change positions gradually. Reduce exercise level temporarily.

Increase fiber and fluid intake. Take the drug with food. Avoid driving after taking the drug. Change positions gradually.

A 13-year-old took a weight loss drug that activated the sympathetic nervous system. Which of the following assessment findings would the nurse expect? Increased cardiac conduction Decreased myocardial contraction Decreased heart rate Increased intranodal conduction time

Increased cardiac conduction

A nurse is caring for a client who is taking naloxone to treat acute morphine toxicity. The nurse should monitor the client for which of the following adverse drug reactions? (Select all) Increased respiratory rate Increased pain Thrombophlebitis Ventricular arrhythmias Hypertension

Increased respiratory rate Increased pain Ventricular arrhythmias Hypertension

6. What information should the nurse include in patient education regarding treatment with tretinoin? Increased sensitivity to sun requires the use of sunscreen. Improvement may not be noted until 3 to 6 months after therapy begins. Nausea and vomiting may appear initially but will gradually abate. It is safe to apply this drug to an existing sunburn.

Increased sensitivity to sun requires the use of sunscreen.

A 3-year-old male is diagnosed with Kawasaki disease. What is the most likely cause of Kawasaki disease? Inflammation of the small capillaries, arteries, and veins Autoimmune injury to the lymphatic vessels A genetic defect causing left heart failure Infectious pericarditis

Inflammation of the small capillaries, arteries, and veins

9. A patient who is taking nitrofurantoin is experiencing muscle weakness, tingling sensations, and numbness in the fingers. What should the nurse tell this patient to do? Take the medication at bedtime to decrease these side effects. Increase fluid intake to decrease these side effects. Inform the health care provider immediately. Decrease the medication to half the regular dose.

Inform the health care provider immediately.

2. What is the primary effect of combination oral contraceptives? Change the vaginal pH Thin cervical mucus Inhibit ovulation Destroy the ovum

Inhibit ovulation

A nurse is caring for a client who is about to begin taking aspirin to reduce the risk of a cardiovascular event. The nurse should identify that the drug inhibits platelet aggregation by which of the following mechanisms? Activating thromboxane A2 Blocking adenosine diphosphate receptor agonists Suppressing specific clotting factors Inhibiting cyclooxygenase action in platelets

Inhibiting cyclooxygenase action in platelets

1. The nurse identifies what parts of the body as common sites of worm infestation? (Select all that apply.) Central nervous system Lungs Intestines Lymphatic system Blood vessels

Intestines Lymphatic system Blood vessels

A 35-year-old who was severely burned is now demonstrating symptomology associated with acute tubular necrosis (ATN). Which form of renal failure is this patient experiencing? Prerenal Extrarenal Intrarenal Postrenal

Intrarenal

1. In counseling a patient about birth control, the nurse would indicate that which form is contraindicated in a patient with multiple sexual partners? Levonorgestrel Intrauterine devices Oral contraceptives Spermicidal jelly and condoms

Intrauterine devices

When a patient is diagnosed with coronary artery disease, the nurse assesses for myocardial: Ischemia Hypertrophy Necrosis Inflammation

Ischemia

A client is taking rifampin to treat TB. The nurse should expect which of the following drugs to be prescribed to the client to prevent possible resistance to rifampin. Select all Gentamicin Vancomycin Isoniazid Metronidazole

Isoniazid

5.How would the nurse best describe the function of activated charcoal in the treatment of poisoning? It binds the toxic substances and induces vomiting in the patient. It binds toxicants irreversibly and renders them nonpoisonous. It stimulates peristalsis and hastens bowel evacuation of toxins. It enhances the activity of antidotes.

It binds toxicants irreversibly and renders them nonpoisonous.

A client is about to begin taking isoniazid to teat TB. The nurse should instruct the client to report which of the following adverse affects? Select all Jaundice Numbness of the hands Dizziness Hearing loss Oral ulcers

Jaundice Numbness of the hands Dizziness

Which type of nephron determines the concentration of the urine? Midcortical Cortical Juxtamedullary Medullary

Juxtamedullary

9.The nurse recognizes what uses of ivermectin? (Select all that apply.) Lice infestation Mite infestation Nematode infestation Tapeworm infestation

Lice infestation Mite infestation Nematode infestation

5. What should the nurse monitor in the patient taking albendazole for nematode infestation? Hearing Urine output Liver enzymes Blood pressure

Liver enzymes

A 75-year-old obese female presents to her primary care provider reporting edema in the lower extremities. Physical exam reveals that she has varicose veins. Upon performing the history, which of the following is a possible cause for the varicose veins? Long periods of standing Extreme exercise Trauma to the deep veins Ischemia

Long periods of standing

The components of the nephron include the: (select all that apply) Loop of Henle Renal Corpuscle Proximal tubule Renal pelvis Convoluted tubule

Loop of Henle Renal Corpuscle Proximal tubule Convoluted tubule

A 20-year-old female is being admitted to the hospital with fever and septic shock. Which set of assessment findings would the nurse expect the patient to exhibit? Low blood pressure and tachycardia Reduced cardiac output, increased systemic vascular resistance, moist cough Severe respiratory distress, jugular venous distention, chest pain Bradycardia, palpitations, confusion, truncal rash

Low blood pressure and tachycardia

A nurse is caring for a client who has a new prescription for dobutamine. The nurse should clarify the prescription with the provider if the client is receiving which of the following types of drugs? (Select all) ACE inhibitor MAOI General anesthetic Tricyclic antidepressant Beta blocker

MAOI General anesthetic Tricyclic antidepressant Beta blocker

1. What is (are) the drug(s) of choice for lice? (Select all that apply.) Lindane Crotamiton Malathion 5% permethrin

Malathion 5% permethrin

10. What is the drug of choice for both East African and West African trypanosomiasis (sleeping sickness)? Sumarin sodium Eflornithine Melarsoprol Nifurtimox

Melarsoprol

3. What medications can be given by the nurse during an intravenous infusion of amphotericin B if the patient experiences rigors? Acetaminophen and diazepam Morphine and diazepam Fentanyl and diazepam Meperidine and dantrolene

Meperidine and dantrolene

A nurse is planning care for a client who has started taking prednisone. Which of the following interventions should the nurse include? Monitor the client's blood glucose. Administer an antacid 30 min prior to prednisone. Administer aspirin rather than NSAIDs if the client has pain. Monitor the client for hyperkalemia.

Monitor the client's blood glucose.

7. Which statement by the nurse best describes the required treatment for pelvic inflammatory disease (PID)? Short-term therapy with one antibiotic is usually enough to provide a cure. Single organisms are usually involved and require single antibiotic therapy. Long-term therapy with multiple antibiotics usually is ineffective. Multiple organisms usually are involved and require multiple antibiotics and treatment regimens.

Multiple organisms usually are involved and require multiple antibiotics and treatment regimens.

A nurse is providing teaching to a client who is taking simvastatin. The nurse should instruct the client to report which of the following manifestations as an indication of a serious adverse reaction that could require discontinuing drug therapy? Bronchoconstriction Muscle pain Lip numbness Somnolence

Muscle pain

A 75-year-old male has severe chest pain and dials 911. Based upon the lab findings indicating a patient has elevated levels of cardiac troponins I and T, the nurse suspects which of the following has occurred? Myocardial infarction (MI) Angina Orthostatic hypotension Raynaud disease

Myocardial infarction (MI)

A nurse is caring for a client who is taking atorvastatin and has a new prescription for gemfibrozil. The nurse should recognize that this drug combination places the client at an increased risk for which of the following adverse effects? Myopathy Hypoglycemia Irregular pulse Kidney failure

Myopathy

A nurse is caring for a client who has a new prescription for propranolol to treat a tachydysrhythmia. The nurse should instruct the client to avoid taking which of the following types of over the counter drugs while taking propranolol? Antihistamines Potassium supplements NSAIDs Vitamin C

NSAIDs

A nurse is caring for a client who has a new prescription for butorphanol. The nurse should monitor the client for which of the following adverse drug reactions? (Select all) Infection Nausea Tachycardia Dizziness Headache

Nausea Dizziness Headache

When discussing the functional unit of the kidney, what term should the nurse use? Collecting duct Pyramid Nephron Calyx

Nephron

4. In preparing an educational program about oral contraceptives (OCs), the nurse includes what information about cancer risks? (Select all that apply.) OCs protect against ovarian cancer. OCs protect against endometrial cancer. OCs protect against cervical cancer. OCs increase the risk of cervical cancer. OCs increase the risk of breast cancer. OCs promote the growth of existing breast cancer.

OCs protect against ovarian cancer. OCs protect against endometrial cancer. OCs promote the growth of existing breast cancer.

A nurse is caring for a client who is taking spironolactone to treat hypertension. The nurse should recognize that which of the following client laboratory values requires immediate intervention? Sodium 140 mEq/L Potassium 5.2 mEq/L Chloride 100 mEq/L Magnesium 1.9 mEq/L

Potassium 5.2 mEq/L

A 40-year-old pregnant woman is concerned about the health of her unborn child. She has phenylketonuria (PKU) and recently had a viral infection. If the child is born with a congenital heart defect, which of the following is the most likely based on the mother's history? Patent ductus arteriosus Tetralogy of Fallot Ventricular septal defect Pulmonic stenosis

Patent ductus arteriosus

8. What is the drug of choice for treatment of syphilis? Erythromycin Penicillin G Azithromycin Vancomycin

Penicillin G

A nurse should identify that clopidrogel is contraindicated for clients who have which of the following conditions? Myocardial infarction Peptic ulcer disease Pancreatitis Myasthenia gravis

Peptic ulcer disease

4. What is the most appropriate treatment for taeniasis (beef or pork tapeworm infestation)? Ivermectin Good handwashing Elimination of meat from the diet Praziquantel

Praziquantel

7. What is a contraindication to treatment with isotretinoin? Diabetes mellitus Pregnancy Hypertension Renal failure

Pregnancy

7. What are contraindications to routine vaccination against smallpox? History of frequent dermatitis syndrome Pregnancy or breastfeeding Occupational contact with animal hides or bones Occasional contact with a person who is immune compromised

Pregnancy or breastfeeding

Blood flow is regulated by five principles - resistance, velocity, laminar versus turbulent flow, and vascular compliance. What is the fifth?

Pressure

A nurse should identify that which of the following drugs is used only for the short term - treatment of a cardiac dysrhythmia because of the serious adverse effects associated with long term use? Quinidine Procainamide Nitroglycerin Verapamil

Procainamide

2. Which at-risk group(s) should be vaccinated against anthrax? Children and pregnant patients Older adult and immunocompromised patients Professionals who handle animal products All health care professionals

Professionals who handle animal products

When describing the male urinary anatomy, which information should the nurse include? The portion of the male urethra that is closest to the bladder is the _____ portion. Membranous Cavernous Vas deferens Prostatic

Prostatic

When a 42-year-old is diagnosed with chronic renal failure, which dietary restriction will the nurse discuss with the patient? Proteins Complex carbohydrates Sugars Fats

Proteins

A nurse is caring for a client who is taking a diuretic. The nurse should instruct the client to include which of the following foods in their diet to increase potassium intake? Raisins Cabbage Cheese Eggs

Raisins

A nurse is monitoring a client who is undergoing anticoagulant therapy with heparin. Which of the following findings should the nurse identify as a possible indication of hemorrhage? Rapid pulse Yellowing of the sclera Elevated blood pressure Pale-colored stools

Rapid pulse

A nurse is caring for a client who is about to begin captopril therapy. Which of the following adverse effects should the nurse instruct the client to report because it can indicate a need to discontinue drug therapy. (Select all) Rash Distorted taste Swelling of the tongue Photosensitivity Dry cough

Rash Distorted taste Swelling of the tongue Dry cough

A nurse is caring for a patient taking aztreonam to treat a respiratory infection. Which of the following health conditions should the nurse recognize as requiring cautious use for this prescription? Glaucoma Closed-head injury Heart failure Renal impairment

Renal impairment

What structure does urine pass through prior to entering the ureters? Urethra Renal pelvis Collecting duct Major calyx

Renal pelvis

When considering pyelonephritis, where is the site of the infection? Renal pelvis Bladder Renal tubules Glomerulus

Renal pelvis

A nurse is teaching a client who has a new prescription for gemfibrozil. Which of the following instructions should the nurse include? (Select all) Take the drug with food. Report any new intolerance to fried foods. Report muscle tenderness. Expect periodic liver function testing. Take the drug once each day.

Report any new intolerance to fried foods. Report muscle tenderness. Expect periodic liver function testing.

A nurse is caring for a client who has a depressive disorder and requires a prescription drug to treat hypertension. The nurse should recognize that which of the following antihypertensive drugs is contraindicated for this client? Reserpine Captopril Hydralazine Eplerenone

Reserpine

1. The nurse understands that infection with Bacillus anthracis causes what? (Select all that apply.) Respiratory illness Dermatologic infection Gastrointestinal illness Cardiovascular system

Respiratory illness Dermatologic infection Gastrointestinal illness

A nurse is caring for a client who is about to begin taking folic acid to treat megaloblastic anemia. The nurse should monitor which of the following laboratory values to determine therapeutic effectiveness? Amylase level Reticulocyte count C-reactive protein Creatinine clearance

Reticulocyte count

Many valvular stenosis and regurgitation disorders in adults have a common etiology. Which of the following conditions should alert the nurse that the patient may have both types of valve dysfunctions? Rheumatic fever or heart disease Connective tissue disorders Syphilis infection Heart failure

Rheumatic fever or heart disease

6. The nurse recognizes that which medication can decrease the effectiveness of oral contraceptives? Rifampin Acetaminophen Multivitamins Furosemide

Rifampin

9. The nurse recognizes that which statement related to SPF is most accurate? SPF is an index of protection against ultraviolet B (UVB) radiation. Individuals who burn easily should use a low SPF sunscreen. UVA and UVB rays are blocked by an SPF of 20 or greater. The relationship between sun protection and SPF is linear.

SPF is an index of protection against ultraviolet B (UVB) radiation.

A nurse is caring for a client who has a new prescription for verapamil. The nurse should clarify the prescription with the provider if the client has a history of which of the following conditions? Migraine headaches Pancreatitis Second-degree AV block Angina pectoris

Second-degree AV block

A nurse is reviewing the medical record of a client who has a new prescription for tramadol. The nurse should identify that which of the following conditions is a contraindication for tramadol? Hyperthyroidism Seizure disorder Rheumatoid arthritis Urinary incontinence

Seizure disorder

10. What would the nurse consider when treatment choices for psoriasis are discussed? Duration of outbreaks Location of lesion Severity of symptoms Patient preference

Severity of symptoms

A 72-year-old female has a history of right heart failure caused by a right ventricular myocardial infarction. Which of the following symptoms are specifically related to her right heart failure? Significant edema to both lower legs and feet Hypertension Decreased urine output Dyspnea upon exertion

Significant edema to both lower legs and feet

A nurse is reviewing prescriptions with a client who has heart disease. The nurse should instruct the client that which of the following drugs is prescribed to treat hypercholesterolemia? Simvastatin Furosemide Losartan Nitroglycerin

Simvastatin

A 50-year-old received trauma to the chest that caused severe impairment of the primary pacemaker cells of the heart. Which of the following areas received the greatest damage? Sinoatrial (SA) node Atrioventricular (AV) node Bundle of His Ventricles

Sinoatrial (SA) node

2. How would the nurse best describe the therapeutic actions of keratolytic agents? Sloughing of skin cells Reducing inflammation and itching Bleaching melanin spots Moisturizing skin

Sloughing of skin cells

A patient is diagnosed with coronary artery disease. Which of the following modifiable risk factors would the nurse suggest the patient change? Smoking cigarettes Drinking tomato juice Living arrangements Eating meat

Smoking cigarettes

7. Which drugs are most commonly used to alter urinary pH to facilitate renal clearance of toxins? Sodium bicarbonate Ammonium chloride Hydrogen peroxide Salicylic acid

Sodium bicarbonate

A nurse is teaching a client who has a new prescription for allopurinol. The nurse should instruct the client to report which of the following adverse drug reactions? (Select all) Palpitations Sore throat Vertigo Bruising Vision changes

Sore throat Vertigo Bruising Vision changes

Which valvular condition is characterized by the valve opening being constricted and narrowed, causing the valve leaflets, or cusps, to fail to open completely? Stenosis Regurgitation Incompetence Insufficiency

Stenosis

A nurse is administering cefotetan IV to a client to treat bacterial meningitis. The nurse notes that the IV insertion site is warm, edematous, and painful to the touch. Which of the following actions should the nurse take? Decrease the rate of the cefotetan infusion. Administer diphenhydramine to the client. Request a prescription for another antibiotic. Stop the cefotetan infusion.

Stop the cefotetan infusion.

4. What is the treatment of choice for tularemia infection? Streptomycin or gentamicin Gentamicin or vancomycin Ciprofloxacin or carbapenem Doxycycline or metronidazole

Streptomycin or gentamicin

A 75-year-old reports loss of urine with cough, sneezing, or laughing. These symptoms support which diagnosis? Urge incontinence Overflow incontinence Stress incontinence Functional incontinence

Stress incontinence

A newborn experiences frequent periods of cyanosis, usually occurring during crying or after feeding. Which cardiac diagnosis does this history support? Tetralogy of Fallot Atrioventricular canal (AVC) defect Ventricular septal defect (VSD) Atrial septal defect (ASD)

Tetralogy of Fallot

A nurse is reviewing the medical record of a client who has a new prescription for celecoxib. The nurse should identify that which of the following conditions is a contraindication to celecoxib? Rheumatoid arthritis Ankylosing spondylitis Sulfonamide allergy Adrenocortical insufficiency

Sulfonamide allergy

8.Which statement best describes the effectiveness of sunscreens in preventing skin cancer? There is no protection from serious skin cancers. All types are prevented by careful use of sunscreen. Sunscreen provides some protection against skin cancers. The protection depends on the strength of the sunscreen.

Sunscreen provides some protection against skin cancers.

A patient who is taking imipenem (primaxin) to treat a bacterial infection contacts the health care professional to report an inability to eat because of mouth pain. Recognizing the AE of imipenem, the HCP should suspect which of the following? Malabsorption Superinfection Anorexia Dental caries

Superinfection

A 50-year-old male with a 30-year history of smoking was diagnosed with bronchogenic cancer. He developed edema and venous distention in the upper extremities and face. Which of the following diagnosis will the nurse observe on the chart? Superior vena cava syndrome (SVCS) Chronic venous insufficiency Thromboembolism Deep vein thrombosis

Superior vena cava syndrome (SVCS)

1. What is the most important step in managing acute poisoning? Identification of the poison Promotion of poison removal Prevention of further absorption Supportive care

Supportive care

A nurse is caring for a client who is about to begin taking itraconazole to treat a fungal infection. The nurse should instruct the client to report which of the following adverse effects of the drug? Tingling in the hands and feet Joint pain Swelling of hands or feet Excessive sweating

Swelling of hands or feet

A patient is diagnosed with orthostatic hypotension. Which of the following symptoms would most likely be reported? Syncope and fainting Headache and blurred vision Chest pain and palpitations Nausea and vomiting

Syncope and fainting

A nurse takes an adult patient's blood pressure and determines it to be normal. What reading did the nurse obtain? Systolic pressure less than 120 mm Hg and diastolic pressure less than 80 mm Hg Systolic pressure between 140 and 150 mm Hg Systolic pressure less than 100 mm Hg regardless of diastolic pressure Systolic pressure greater than 140 mm Hg and a diastolic pressure of 100 mm Hg

Systolic pressure less than 120 mm Hg and diastolic pressure less than 80 mm Hg

A patient who is taking ciprofloxacin to treat a respiratory tract infection contacts the health care professional to report dyspepsia. The HCP should recommend which of the following instructions? Take an antacid at least 2 hr before taking the drug. Take the drug with a cup of coffee. Take an iron supplement with the drug. Take the drug with 240 mL (8 oz) of milk.

Take an antacid at least 2 hr before taking the drug.

Hallmark features of a Streptococcal sore throat include (you may chose more than one): Tender lymph nodes Scarlet fever rash Hoarseness and cough Runny nose Abdominal pain

Tender lymph nodes Scarlett fever rash Abdominal pain

A health care professional is caring for a patient who is taking ciprofloxacin to treat a UTI and has rheumatoid arthritis, for which he takes prednisolone (prelone). Recognizing the adverse effects of ciprofloxacin, the health care professional should tell the pt to report which of the following? Tachycardia Hair loss Insomnia Tendon pain

Tendon pain

A nurse is caring for a client who has a glomerular filtration rate of 10 mL/ min and a reduced urine output. The nurse should clarify a prescription for hydrochlorothiazide for this client because of which of the following characteristics of the drug? The drug can cause hypoglycemia in clients who have a low urine output. The drug does not reduce blood pressure for clients who have a low urine output. The drug can increase the risk of pulmonary edema for clients who have renal insufficiency. The drug does not promote diuresis for clients who have renal insufficiency.

The drug does not promote diuresis for clients who have renal insufficiency.

Which structures would not receive an electrical impulse when the patient is experiencing a left bundle branch block? The left ventricle Bundle of His Sinoatrial (SA) node Atrioventricular (AV) node

The left ventricle

A nurse is caring for a client who is receiving morphine to relieve severe pain. The nurse should monitor the client for which of the following adverse drug reactions? (Select all) Diarrhea Urinary retention Respiratory depression Sedation Orthostatic hypotension

Urinary retention Respiratory depression Sedation Orthostatic hypotension

The onset of anaphylactic shock is usually: sudden and life threatening. mild delayed by several hours. delayed by 24 hours.

sudden and life threatening.

The blood vessels of the kidneys are innervated by the: somatic nervous system sympathetic nervous system parasympathetic nervous system vagus nerve

sympathetic nervous system

The trigone is defined as: the inner area of the kidney a triangular area between the openings of the two ureters and the urethra the orifice of the urete the three divisions of the loop of Henle.

a triangular area between the openings of the two ureters and the urethra

6. What is the statement that best describes smallpox? It is transmitted by contamination from infected livestock. The only natural reservoir for the virus is humans. The fatality rate is relatively low because it is not especially contagious. It is a weak threat as a biologic weapon because of vaccination programs.

The only natural reservoir for the virus is humans.

5. While reviewing a patient's history, the nurse identifies what finding as a contraindication to the administration of oral contraceptives? The patient's sister has breast cancer. The patient has a history of thrombophlebitis. The patient is 3 months postpartum and is breastfeeding. The patient's mother had a cerebrovascular accident (CVA) at age 65.

The patient has a history of thrombophlebitis.

A nurse is caring for a client who is scheduled for an outpatient surgical procedure and reports taking aspirin 81 mg daily, including this morning. The nurse should identify that this places the client at risk for which of the following complications? Uncontrolled bleeding Myocardial infarction Respiratory depression Decreased renal perfusion

Uncontrolled bleeding

9. What does the nurse recognize as the risk of radioactive fallout? Illness begins within 24 hours and lasts as long as 2 years. There is a delayed risk of thyroid cancer. If no symptoms appear within 1 month, no illness will occur. It may take as long as 1 week after exposure for symptoms to appear.

There is a delayed risk of thyroid cancer.

7. The nurse correlates which action with the contraceptive effects of the minipill? Blocks implantation Thickens cervical mucus Changes vaginal pH Inhibits ovulation

Thickens cervical mucus

A nurse is caring for a client who has a new diagnosis of bacterial meningitis. The nurse should expect the providers to prescribe a drug from which of the following classifications? First generation cephalosporins Third generation cephalosporins Monobactams Macrolides

Third generation cephalosporins

A nurse is caring for a client who is taking clopidogrel to precent stent restenosis. The nurse should monitor the client for which of the following adverse reactions? Hyperuricemia Hyponatremia Lymphocytopenia Thrombocytopenia

Thrombocytopenia

What term should the nurse use to document a detached blood clot? Thromboembolus Thrombus Embolus Infarction

Thromboembolus

3. Which clinical manifestation can be an adverse effect of topical salicylic acid? Sedation Tinnitus Diarrhea Nausea

Tinnitus

A nurse is caring for a client who asks about taking acetaminophen. The nurse should identify that acetaminophen is indicated for which of the following conditions? (Select all) To reduce fever To decrease inflammation To relieve mild pain To promote sedation To alleviate anxiety

To reduce fever To relieve mild pain

10. The nurse would provide what teaching to the patient with genital herpes? (Select all that apply.) Take the medication faithfully to eradicate the virus. Use a condom at all times for sex to decrease the risk of transmission. The virus cannot be transmitted if there are no open vesicles on the skin. A recurrence can happen at any time throughout life.

Use a condom at all times for sex to decrease the risk of transmission. A recurrence can happen at any time throughout life.

A nurse is providing teaching for a client taking an oral contraceptive and is about to begin rifampin therapy to treat TB. Which of the following instructions should the nurse include? Increase the rifampin dose. Increase the oral contraceptive dose. Allow 2 hr between taking the two drugs. Use a non-hormonal form of contraception.

Use a non-hormonal form of contraception.

3. What is the most reliable method of identifying poison consumed by a patient? Asking the patient Asking the patient's family members Inspecting the stomach contents Using chromatographic or spectrometric studies of body fluids

Using chromatographic or spectrometric studies of body fluids

A nurse is caring for a patient whose sputum culture results indicate MRSA. The client will most likely be administered Trimethoprim/sulfamethoxazole Tetracycline Cephalexin Vancomycin

Vancomycin

What is the nurse monitoring when observing the QRS complex on the electrocardiogram? Ventricular activity Pulmonary artery closure Mitral valve opening Aortic valve closing

Ventricular activity

Intrauterine exposure to which factor could be responsible for a diagnosis of congenital heart disease? Viral infection Dextroamphetamine Diabetes Alcohol exposure

Viral infection

5. The nurse identifies topical retinoids as derivatives of what? Vitamin A Vitamin D Salicylic acid Arachidonic acid

Vitamin A

A nurse should assess a client who has a megaloblastic anemia for indications of which of the following vitamin deficiencies? Vitamin C Vitamin K Vitamin B12 Vitamin D

Vitamin B12

A nurse in an emergency department is assessing a client who has been taking warfarin and is experiencing rectal bleeding. Which of the following drugs should the nurse expect to administer to the client? Filgrastim Deferoxamine Protamine Vitamin K

Vitamin K

2. Which effects may be the result of vomiting, diarrhea, or sweating? Fatigue and confusion Volume depletion Gastrointestinal irritation Alteration of oxygen saturation

Volume depletion

A nurse is caring for a client who is opioid dependent and has a new prescription for butorphanol. The nurse should monitor the client for which of the following manifestations of abstinence syndrome? (Select all) Bronchospasm Vomiting Peripheral edema Abdominal cramps Hypertension

Vomiting Abdominal cramps Hypertension

A nurse is reviewing the drug list for a client who has a new prescription for allopurinol. The nurse should identify that which of the following drugs interact with allopurinol? Warfarin Ibuprofen Insulin Furosemide

Warfarin

A nurse is caring for a client who has a prescription for aspirin to treat an ankle sprain. The nurse should instruct the client to report which of the following adverse drug reactions? Polyuria Bone pain Weight gain Infection

Weight gain

You are caring for a 45- year old patient who has fully recovered from COVID-19 6 months ago. They have no know cardiac risk factors or history of heart disease. They are now experiencing extreme fatigue and some rhythm abnormalities. Could the previous COVID-19 infection be now causing these problems? Yes No

Yes

The pericardium is: a membranous sac that encloses the heart the outer muscular layer of the heart the innermost layer of the heart chambers the heart's fibrous skeleton

a membranous sac that encloses the heart

A 30-year-old male is demonstrating hematuria with red blood cell casts and proteinuria exceeding 3-5 grams per day, with albumin being the major protein. The most probable diagnosis the nurse will see documented on the chart is: acute glomerulonephritis chronic pyelonephritis Cystitis renal calculi

acute glomerulonephritis

A 30-year-old male is demonstrating hematuria with red blood cell casts and proteinuria exceeding 3-5 grams per day, with albumin being the major protein. The most probable diagnosis the nurse will see documented on the chart is: chronic pyelonephritis cystitis renal calculi acute glomerulonephritis

acute glomerulonephritis

A 30-year-old female presents to her primary care provider with fever, cardiac murmur, and petechial skin lesions and is diagnosed with infective endocarditis. The most likely cause of the disease is: bacteria. viruses. fungi. parasites.

bacteria.

Which vessels have the least resistance, capillaries or arterioles?

capillaries

During the ventricular ejection phase, the mitral valve is normally open or closed?

closed

The renin-angiotensin system will be activated by: increased blood volume decreased blood pressure in the afferent arterioles elevated sodium concentrations renal hypertension

decreased blood pressure in the afferent arterioles

The cardiac electrical impulse normally begins spontaneously in the sinoatrial (SA) node because it: depolarizes more rapidly than other automatic cells of the heart. has rich sympathetic innervation via the vagus nerve. is the only area of the heart capable of spontaneous depolarization. has a superior location in the right atrium.

depolarizes more rapidly than other automatic cells of the heart.

The internal lining of the cardiovascular system is formed by what tissue? endothelium tunica adventitia connective mesothelium

endothelium

A 10-year-old male presents with fever, lymphadenopathy, arthralgia, and nosebleeds. He is diagnosed with rheumatic heart disease. The most likely cause of this disease is: group A beta-hemolytic streptococcus infections. human immunodeficiency virus (HIV) infections. congenital heart defects. acute pericarditis.

group A beta-hemolytic streptococcus infections.

Acute poststreptococcal glomerulonephritis is primarily caused by: swelling of mesangial cells in the Bowman space in response to the presence of bacteria. immune complex deposition in the glomerular capillaries and inflammatory damage. inflammatory factors that stimulate cellular proliferation of epithelial cells. accumulation of antiglomerular basement membrane antibodies.

immune complex deposition in the glomerular capillaries and inflammatory damage.

The thoracic lymphatic duct drains into the: left subclavian vein. left subclavian artery. right atrium. right subclavian vein.

left subclavian vein.

Which chamber of the heart generates the highest pressure? left ventricle right ventricle right atrium left atrium

left ventricle

A nurse recalls the most common cardiac valve disease in the United States is: mitral valve prolapse. tricuspid valve prolapse. pulmonary stenosis. aortic stenosis.

mitral valve prolapse.

When a person is in shock, a nurse remembers impairment in cellular metabolism is caused by: inadequate tissue perfusion. lack of nervous or endocrine stimulation. release of toxic substances. free radical formation.

nadequate tissue perfusion.

The PR interval viewed on a normal electrocardiogram represents: onset of atrial activation to onset of ventricular activity "electrical systole" of the ventricles ventricular depolarization. atrial depolarization.

onset of atrial activation to onset of ventricular activity

The glomerular filtration rate (GFR) is directly related to the: oncotic pressure in the glomerular capillaries vascular resistance in the glomerular arterioles hydrostatic pressure in the Bowman's capsule perfusion pressure in the glomerular capillaries

perfusion pressure in the glomerular capillaries

A patient with left heart failure starts to have a cough and dyspnea. Pulmonary symptoms common to left heart failure are a result of: pulmonary vascular congestion. bronchoconstriction. inflammatory pulmonary edema. decreased cardiac output.

pulmonary vascular congestion.

A 5-year-old female is found to have hypertension during three separate visits to her primary care provider. The nurse would expect tests to suggest that the hypertension is secondary to: renal disease brain tumor hyponatremia hypocalcemia

renal disease

A HCP is caring for a pt who is about to begin taking metronidazole (flagyl) to treat an anaerobic intra-abdominal bacterial infection. The HCP should recognize that cautious use of the drug is indicated if the patient also has which of the following? Seizure disorder Hearing loss Asthma Anemia

seizure disorder

A patient most prone to multiple organ dysfunction syndrome (MODS) is a patient with: septic shock autoimmune disease. myocardial infarction (MI). pulmonary disease.

septic shock

A 52-year-old male presents with pooling of blood in the veins of the lower extremities and edema. The diagnosis is chronic venous insufficiency, and an expected assessment finding of this disorder is: skin hyperpigmentation edema above the knee. gangrene deep vein thrombus formation

skin hyperpigmentation

3. The nurse understands that ________ poses the greatest risk for thromboembolic phenomena in a patient taking oral contraceptives. high intake of saturated fats sedentary lifestyle low body weight smoking

smoking

3. The nurse identifies __________ as a common type of worm infestation in the southern United States. trichinosis strongyloidiasis ascariasis Brugia

strongyloidiasis

A nurse realizes glucose will be excreted in the urine when: the maximum rate of glucose filtration is achieved glucose is consumed the carrier molecules have reached their maximum the ability of the kidneys to regulate blood glucose is lost

the carrier molecules have reached their maximum

Bone fractures are a risk factor in chronic renal failure because: autoantibodies to calcium molecules develop calcium is lost in the urine the kidneys fail to activate vitamin D osteoblast activity is excessive

the kidneys fail to activate vitamin D

When a nurse is checking the urinalysis, plasma proteins should be absent from the urine because: all filtered proteins are subsequently reabsorbed the negative charge of the glomerular filtration membrane repels the plasma proteins the texture of the covering surrounding plasma proteins interferes with reabsorption filtered proteins are subsequently degraded before elimination

the negative charge of the glomerular filtration membrane repels the plasma proteins

As a result of blockage in the pulmonary artery, blood would first back up into the: the right ventricle aorta pulmonary veins left ventricle

the right ventricle

The most common condition associated with the development of acute pyelonephritis is: urinary tract obstruction nephrotic syndrome Cystitis Renal cancer

urinary tract obstruction

A nurse is caring for a patient about to begin gentamicin for an infection. The nurse should monitor the client for an alteration in Bowel function Peripheral pulses Urine output Level of consciousness

urine output

While planning care for a patient with renal calculi, the nurse remembers the most important factor in renal calculus formation is: body temperature gender urine pH serum mineral concentrations

urine pH

Which vessels have the greatest compliance - the ability to accommodate an increase in volume for a given increase in pressure? Veins or arteries?

veins

The descending segment of the loop of Henle primarily allows for: sodium secretion water reabsorption potassium secretion hydrogen ion reabsorption

water reabsorption

The nurse teaches a patient about Camila. Which statement by the patient requires an intervention by the nurse? "I might have irregular bleeding while taking this pill." "These pills do not usually cause blood clots." "I should take this pill at the same time every day." "This pill works primarily by preventing ovulation."

"This pill works primarily by preventing ovulation."

Which statement about botulinum toxin does the nurse identify as true? A.Death from botulinum toxin results from paralysis of the muscles of respiration. B.Botulinum antitoxin can reverse damage that has already occurred. C.No immunoglobulin is available for botulinum toxin. D.Botulinum toxin acts on the cholinergic nerve terminals to enhance acetylcholine release.

A

10. The nurse understands that methenamine is contraindicated in which patient? A 40-year-old patient with diabetes A 50-year-old patient with liver failure A 50-year-old patient with myasthenia gravis A 60-year-old patient with a history of asthma

A 50-year-old patient with liver failure

2. In which patient should tinidazole be used with caution? A 50 year old with diabetes A 60 year old with meningitis A 30 year old with hypertension A 45 year old with hyperthyroidism

A 60 year old with meningitis

The nurse instructs a patient in the use of combination oral contraceptives for birth control. The nurse determines that teaching is successful if the patient makes which statement? A."I'll avoid herbal products such as St. John's wort." B."Birth control pills don't have serious side effects." C."I can continue taking birth control before elective surgeries." D."I should take the pill with food to prevent an upset stomach."

A."I'll avoid herbal products such as St. John's wort."

A patient receiving chemotherapy is anemic. Which medication does the nurse anticipate administering to this patient? A.Filgrastim B.Pegfilgrastim C.Neumega D.Erythropoietin

A.Erythropoietin

Which statement will the nurse include when teaching a patient with HIV about management of the disease? A.HIV is considered a chronic disease. B.HIV infection can be cured with 1 year of therapy. C.Drug resistance does not occur in HIV treatment. D.Blood cultures and red blood cell counts are the principal laboratory tests to guide HIV treatment protocols.

A.HIV is considered a chronic disease.

A patient has used a topical glucocorticoid preparation daily for several years. It is most important for the nurse to assess the patient for which adverse effect of long-term topical glucocorticoid therapy? Shedding of dry scales Open and closed comedones Atrophy of dermal and epidermal layers Cancerous lesions

Atrophy of dermal and epidermal layers

2. Which of the following skin conditions is benign? A.Kaposi sarcoma B.Actinic keratosis C.Basal cell carcinoma D.Cutaneous melanoma

B

A 22-year-old female patient is prescribed isotretinoin for severe acne vulgaris. What statement will the nurse include in patient teaching? A.Vitamin A supplements can be used with this medication. B.Pregnancy must be avoided while taking this medication. C.Oral contraceptives must not be used with this medication. The most common adverse effect of this medication is depression

B

A patient contacts a clinic nurse to determine the proper action after she forgot to take her oral contraceptive [Ortho Tri-Cyclen] for the past 2 days during the first week of a 28-day regimen. Which response by the nurse is most appropriate? A."Take the omitted two doses together with the next dose." B."Take two doses per day on the following 2 days." C."Stop taking the oral contraceptive until menstruation occurs." D."Take a dose now and continue with the scheduled doses."

B."Take two doses per day on the following 2 days."

Which patient does the nurse identify as most likely to need treatment with trimethoprim/sulfamethoxazole [Bactrim] for a period of 6 months? A.A female patient with acute pyelonephritis B.A male patient with acute prostatitis C.A female patient with recurring acute urinary tract infections D.A male patient with acute cystitis

B.A male patient with acute prostatitis

6. What is a drug of choice for American trypanosomiasis (Chagas' disease)? Iodoquinol Benznidazole Nitazoxanide Pyrimethamine

Benznidazole

A patient is prescribed metronidazole [Flagyl] to treat giardiasis. It is most important for the nurse to teach the patient that while taking this medication, which of the following should be avoided? A.Acetaminophen [Tylenol] B.Milk and dairy products C.Alcoholic beverages D.Sun exposure

C

A 65 year old right-handed male suffered from a stroke involving the left frontal lobe including Broca's area. Based on this information, the nurse knows that the patient: A. Will have deficits in gauging distance B. Will not be able to comprehend what is being said C. Will have difficulty articulating words D. Will not be able to name objects

C. Will have difficulty articulating words

A patient is receiving amphotericin B to treat a systemic fungal infection. To prevent renal damage, it is most important for the nurse to do what? A.Administer the medication through a central venous access device. B.Administer potassium supplements. C.Administer 1000 mL of 0.9% saline. D.Administer the medication orally.

C.Administer 1000 mL of 0.9% saline.

Which of the following is NOT a serious adverse effect of long-term oral glucocorticoid therapy? A.Adrenal suppression B.Osteoporosis C.Hypoglycemia D.Peptic ulcer disease

C.Hypoglycemia

1. The nurse understands that, according to the Centers for Disease Control and Prevention (CDC), there is an increase in which sexually transmitted disease (STD)? (Select all that apply) Clamydia Syphillis Gonorrhea Chancroid

Clamydia Syphillis Gonorrhea

3. What is the preferred treatment of pneumonia infections in infants born to women with cervical Chlamydia trachomatis? Vancomycin Erythromycin Zidovudine Doxycycline

Erythromycin

4. Which bacterium is the most common cause of community-acquired UTIs? Proteus Escherichia coli Klebsiella Enterobacter

Escherichia coli

Which drugs are common treatments of inflammatory skin conditions? Glucocorticoids Nonsteroidal antiinflammatory drugs (NSAIDs) Antihistamines Topical anesthetics

Glucocorticoids

3. Which bacteria are often involved in hospital-acquired UTIs? (Select all that apply.) Klebsiella Proteus Enterobacter Streptococci

Klebsiella Proteus Enterobacter

1. What is the drug of choice for symptomatic intestinal amebiasis and systemic amebiasis? Doxycycline Metronidazole Trimetrexate Suramin

Metronidazole

A patient is diagnosed with C. difficile infection. The nurse anticipates administering which medication? Daptomycin Metronidazole Rifampin Rifaximin

Metronidazole

10. How should the nurse teach a patient to apply ciclopirox? Once every other day, removing the medication on opposite days Once daily, applying new coats over the old ones for the duration of treatment Once daily over the previous dose, removing all of the medication once a week Once in the morning, removing the medication before bedtime

Once daily over the previous dose, removing all of the medication once a week

The patient is being discharged with continued ciprofloxacin therapy. When providing discharge teaching, the nurse should advise the patient to call the healthcare provider immediately if what develops? Pain in the heel of the foot Nausea Diarrhea Headache

Pain in the heel of the foot

A 22-year-old female patient is prescribed isotretinoin for severe acne vulgaris. What statement will the nurse include in patient teaching? Vitamin A supplements can be used with this medication. Pregnancy must be avoided while taking this medication. Oral contraceptives must not be used with this medication. The most common adverse effect of this medication is depression.

Pregnancy must be avoided while taking this medication.

2. What factors are involved in complicated urinary tract infections (UTIs)? (Select all that apply.) Increased urinary output Renal stones Prostatic hypertrophy An indwelling catheter

Renal stones Prostatic hypertrophy An indwelling catheter

8. For what side effects of pentamidine should the nurse monitor the patient during therapy? (Select all that apply.) Severe hypotension Hypoglycemia Hypernatremia Dysrhythmias

Severe hypotension Hypoglycemia Dysrhythmias

8. The nurse should instruct the patient taking nitrofurantoin and complaining of nausea and diarrhea to do what to decrease this side effect? Take the medication at bedtime. Take the medication with meals or milk. Take the medication with a large glass of water. Take the medication with bismuth subsalicylate.

Take the medication with meals or milk.

A patient with increased intracranial pressure due to hydrocephalus has which of the following? A. Increased capacity for cerebral blood flow B. Decreased capacity for cerebral blood flow C. Increased capacity for brain tissue swelling D. Decrease in systolic blood pressure

idk

A patient is prescribed an antibiotic to treat a urinary tract infection. What statement by the patient indicates a need for further teaching? "I can stop the medication as soon as the symptoms have disappeared." "I will drink more fluids to help clear up the infection." "I will stop the medication and contact the doctor if I develop a rash." "I should immediately report vaginal itching or discharge."

"I can stop the medication as soon as the symptoms have disappeared."

The nurse instructs a patient in the use of combination oral contraceptives for birth control. The nurse determines that teaching is successful if the patient makes which statement? "I'll avoid herbal products such as St. John's wort." "Birth control pills don't have serious side effects." "I can continue taking birth control before elective surgeries." "I should take the pill with food to prevent an upset stomach."

"I'll avoid herbal products such as St. John's wort."

9. What should the nurse teach a patient with asthma who is taking inhaled pentamidine? "This medication will not have an impact on your asthma." "You will have to stop your asthma medications when taking this medication." "You may experience cough or bronchospasms, which can be controlled with the use of steroids." "You may experience cough or bronchospasms, which can be controlled with the use of an inhaled bronchodilator."

"You may experience cough or bronchospasms, which can be controlled with the use of an inhaled bronchodilator."

Which statement about syphilis does the nurse identify as true? A.Penicillin G is the drug of choice for all stages of syphilis. B.Early signs of congenital syphilis include purulent discharge from the urethra. C.The risk of neurosyphilis is decreased in individuals with HIV infection. D.Syphilis develops in one abrupt stage.

A.Penicillin G is the drug of choice for all stages of syphilis.

The nurse knows that severe nausea and vomiting from chemotherapy are caused by what? A.Stimulation of the chemoreceptor trigger zone B.Anxiety and anticipatory grieving C.Increased peristalsis of the gastrointestinal tract D.Injury of the epithelial lining of the esophagus and stomach

A.Stimulation of the chemoreceptor trigger zone

A patient with a drug overdose is admitted to the emergency department. The patient is lethargic. The healthcare provider prescribes activated charcoal. How should the nurse should administer the activated charcoal? A.Rectal route as an enema B.Oral administration through a nasogastric tube C.Intravenously through a central catheter D.Intramuscularly, with the injection given into a large muscle

B

3. Which of the following is a symptom of chronic renal failure? A.Hypotension B.Hypokalemia C.Hypocalcemia D.Hypernatremia

C

A patient who sustained second- and third-degree burns has been prescribed mafenide. Which statement about mafenide does the nurse identify as true? A.Use of mafenide can cause alkalosis. B.Mafenide is painful upon application. C.A blue-green to gray discoloration of the skin occurs with mafenide therapy. D.Mafenide exerts its therapeutic effect by the release of free silver.

B.Mafenide is painful upon application.

A patient is receiving chemotherapy. Which laboratory result will most likely require chemotherapy to be withheld? A.Platelet count of 157,000/mm3 B.Neutrophil count of 375/mm3 C.Hemoglobin of 12.2 g/dL D.Partial thromboplastin time of 35 seconds

B.Neutrophil count of 375/mm3

A patient with asthma is prescribed triamcinolone acetonide [Azmacort]. What should the nurse do? A.Take the patient's pulse before administering the medication. B.Teach the patient to use a spacer to prevent a fungal infection. C.Instruct the patient to use this drug to treat an acute attack. D.Encourage the patient to avoid weight-bearing activity.

B.Teach the patient to use a spacer to prevent a fungal infection.

1. Which of the following skin conditions is caused by a bacterial infection? A.Warts B.Varicella C.Impetigo D.Herpes simplex

C

A patient who takes warfarin has been prescribed sulfadiazine. When teaching the patient about this drug, which statement will the nurse include? A."If you become pregnant, it is safe to take sulfadiazine." B."You should limit your fluid intake while taking sulfadiazine." C."Avoid prolonged exposure to sunlight, wear protective clothing, and apply a sunscreen to exposed skin." D."You will most likely need to have an increase in the dose of warfarin while taking sulfadiazine."

C."Avoid prolonged exposure to sunlight, wear protective clothing, and apply a sunscreen to exposed skin."

A patient asks about home treatment for genital warts caused by human papillomaviruses. The nurse should recommend which treatment? A.10% podophyllin [Podofin] B.80% trichloroacetic acid (TCA) C.0.5% podofilox [Condylox] D.Acyclovir [Zovirax]

C.0.5% podofilox [Condylox]

A patient with an isolated frontal lobe traumatic brain injury would be most likely to exhibit which of the following? A. Changes in behavior/personality and altered judgment B. Inability to link objects with past experiences C. Lack of fine body movements and coordination D. Difficulty with equilibrium and balance

I THINK A. Changes in behavior/personality and altered judgment

8. The nurse is caring for a patient receiving amphotericin B. What action should the nurse should take to minimuze kidney damage? Pretreat with dimenhydrinate and dantrolene. Infuse 1 L of saline on the days amphotericin is infused. Dilute amphotericin B with an additional 1 L of saline. Infuse an additional 1500 L of fluid after infusion of amphotericin B.

Infuse 1 L of saline on the days amphotericin is infused.

Which bacteria are often involved in hospital-acquired UTIs? (Select all that apply.) Klebsiella Proteus Enterobacter Streptococci

Klebsiella Proteus Enterobacter

4. Which drug when taken with metronidazole should concern the nurse? Digoxin Furosemide Lithium Potassium supplement

Lithium

It is most important for the nurse to avoid administering oral ciprofloxacin to this patient with which food? Bananas Baked chicken Grapefruit juice Milk

Milk

A patient has been prescribed ciprofloxacin for treatment of a urinary tract infection with Escherichia coli. Before administering the drug, it is most important for the nurse to assess the patient for a history of what? Hypertension Diabetes mellitus Myasthenia gravis Seasonal allergies

Myasthenia gravis

Which statement about superinfections does the nurse identify as true? Superinfections are more common in patients treated with narrow-spectrum drugs. Superinfection is defined as a new infection that appears during the course of treatment for a primary infection. Superinfections are caused by viruses. Superinfections are easy to treat.

Superinfection is defined as a new infection that appears during the course of treatment for a primary infection.

1. The nurse understands that the second most common type of infection encountered today is what? Bacterial endocarditis Bronchitis Urinary tract infection Cellulitis

Urinary tract infection

6. What should the nurse regularly monitor in a patient taking long-term antibiotics for UTIs? Complete blood cell count (CBC) with differential Urine cultures Urine specific gravity Hemoglobin and hematocrit (H & H)

Urine cultures

The nurse identifies topical retinoids as derivatives of what? Vitamin A Vitamin D Salicylic acid Arachidonic acid

Vitamin A

Which patient should receive prophylactic antibiotic therapy? A patient who is to have his teeth cleaned A patient who is scheduled for a hysterectomy A patient with a white blood cell count of 8000 cells/mm3 A patient with a high fever without an identifiable cause

A patient who is scheduled for a hysterectomy

Which patient would be at greatest risk of developing a venous thromboembolism (VTE) if a combination oral contraceptive were prescribed? A 25-year-old patient who drinks 3 to 4 alcoholic drinks a day A 45-year-old patient who has a family history of stroke A 22-year-old patient who smokes 2 packs of cigarettes a day A 29-year-old patient who has used birth control pills for 9 years

A 22-year-old patient who smokes 2 packs of cigarettes a day

6. The nurse understands that unless the person is experiencing a life-threatening illness, which patient should not receive itraconazole? A 40-year-old patient with diabetes A 45-year-old patient with hypertension A 60-year-old patient with heart failure A 30-year-old patient with Raynaud's disease

A 60-year-old patient with heart failure

A patient with scabies is prescribed permethrin [Elimite]. The nurse will administer this medication in which way? A.As an oral solution in three divided doses B.As a topical cream applied to the entire body and rinsed after 8 hours C.By diluting in water and soaking the affected area for 15 minutes D.By spraying the mattress, bed linens, and clothing

B

2. A patient presents with flank pain and anuria followed by polyuria after undergoing catheterization of the ureters. What is the most likely cause of this condition? A.Acute tubular necrosis B.Prerenal acute renal failure C.Postrenal acute renal failure D.Intrarenal acute renal failure

C

A nurse is teaching a child safety course to parents. What instructions should the nurse provide about the initial action for ingestion of a toxic agent? A.The child should be taken to the nearest hospital emergency department. B.The parent should administer syrup of ipecac to induce vomiting. C.The parent should immediately contact the local Poison Control Center. D.The child should drink at least two 8-ounce glasses of water.

C

A patient has used a topical glucocorticoid preparation daily for several years. It is most important for the nurse to assess the patient for which adverse effect of long-term topical glucocorticoid therapy? A.Shedding of dry scales B.Open and closed comedones C.Atrophy of dermal and epidermal layers D.Cancerous lesions

C

A patient is receiving amphotericin B to treat a systemic fungal infection. To prevent renal damage, it is most important for the nurse to do what? A.Administer the medication through a central venous access device. B.Administer potassium supplements. C.Administer 1000 mL of 0.9% saline. D.Administer the medication orally.

C

Routine administration of the anthrax vaccine is recommended for which group of individuals? A.International travelers B.Healthcare workers C.Military personnel D.Postal workers

C

Which patient would be at greatest risk of developing a venous thromboembolism (VTE) if a combination oral contraceptive were prescribed? A.A 25-year-old patient who drinks 3 to 4 alcoholic drinks a day B.A 45-year-old patient who has a family history of stroke C.A 22-year-old patient who smokes 2 packs of cigarettes a day D.A 29-year-old patient who has used birth control pills for 9 years

C.A 22-year-old patient who smokes 2 packs of cigarettes a day

Which patient does the nurse identify as most likely to need treatment with trimethoprim/sulfamethoxazole [Bactrim] for a period of 6 months? A.A female patient with acute pyelonephritis B.A male patient with acute prostatitis C.A female patient with recurring acute urinary tract infections D.A male patient with acute cystitis

C.A female patient with recurring acute urinary tract infections

Which drug does the nurse identify as a urinary tract antiseptic? A.Ciprofloxacin B.Ceftriaxone C.Nitrofurantoin D.Ceftazidime

C.Nitrofurantoin

2. What is the most common cause of bacterial STDs in the United States? Herpes simplex Helicobactor pylori Chlamydia trachomatis Condylomata acuminata

Chlamydia trachomatis

2. The nurse suspects a female patient who suffered a CVA is not able to empty her bladder completely when she urinates. Which test does the nurse anticipate will be ordered? A.Cystometry B.Uroflowmetry C.Video urodynamics D.Postvoid residual urine

D

A nuclear bomb has exploded. Which medication does the nurse anticipate administering to survivors to reduce the risk of thyroid cancer? A.Pentetate zinc trisodium B.Pentetate calcium trisodium C.Prussian blue D.Potassium iodide

D

2. Which medication can be combined with amphotericin B to produce antifungal effects equivalent to those of high-dose amphotericin alone? Griseofulvin Flucytosine Itraconazol Tolnaftate

Flucytosine

You are assigned to care for a patient who was diagnosed with a tumor of the medulla involving cranial nerves IX and X. Based on this information, you would expect to take which of the following actions? A. Teach the patient how to perform a visual scan B. Perform a swallow test before feeding the patient C. Institute fall precautions for the patient with gait ataxia D. Provide information in simple steps to help with memory deficits

I THINK B. Perform a swallow test before feeding the patient

Fluoroquinolones should be discontinued immediately if what happens? Nausea, vomiting, or diarrhea is experienced. Dizziness, headache, or confusion occurs. Tendon pain or inflammation develops. Theophylline is prescribed for asthma.

Tendon pain or inflammation develops.

7. What information should the nurse provide to a patient taking nitrofurantoin for a UTI? The patient should avoid the sun and wear sunscreen. The patient's urine will turn a harmless brown color. The patient will have an increased urine output. The patient will have to increase fluid intake.

The patient's urine will turn a harmless brown color.

5. When would the nurse expect prophylactic antibiotics to be prescribed for a long period for a patient? When the patient has three or more UTIs a year When the patient has a hospital-acquired UTI After the use of an indwelling catheter When the patient has "honeymoon cystitis"

When the patient has three or more UTIs a year

A patient has been prescribed oral ciprofloxacin [Cipro] for a skin infection. When administering the medication, it is most important for the nurse to do what? Monitor for a decrease in the prothrombin time (PT) if the patient is also taking warfarin [Coumadin]. Withhold antacids and milk products for 6 hours before or 2 hours afterward. Inform the healthcare provider if the patient has a history of asthma. Assess the skin for Stevens-Johnson syndrome.

Withhold antacids and milk products for 6 hours before or 2 hours afterward.

7. About what harmless side effect should the nurse warn a patient taking nitazoxanide? Hardening of the soles Red discoloration of the urine Darkening of the skin Yellow discoloration of the sclera

Yellow discoloration of the sclera


Kaugnay na mga set ng pag-aaral

B&G Chptr 8 communication Systems

View Set

ch16: life cycle nutrition: infancy, childhood, and adolescence

View Set

CPH Exam- a mixture of everything

View Set

Arizona Laws and Rules Pertinent to Insurance chapter 18

View Set

ECONS200 Waikato Short Answer June 2019

View Set

Section A: Introduction to Supply Chain

View Set

Ch 3 - Vectors and two dimensional motion

View Set